LSAT and Law School Admissions Forum

Get expert LSAT preparation and law school admissions advice from PowerScore Test Preparation.

 Administrator
PowerScore Staff
  • PowerScore Staff
  • Posts: 8916
  • Joined: Feb 02, 2011
|
#63984
Complete Question Explanation

Must Be True—SN, #%. The correct answer choice is (D)

In this stimulus we are presented with a fact set. To begin, most genetic research advances give rise
to ethical dilemmas.

..... ..... Genetic research :most: give rise to ethical dilemmas

Further, most genetic research is funded solely by the government, the others are funded exclusively
by corporations, and one of these two sources of funding is necessary for genetic research to take
place:

Genetic research projects not funded by the government are funded by corporations:

..... ..... funded by government :arrow: funded by corporations
..... ..... funded by corporations :arrow: funded by government

For any genetic research one or the other source of funding is necessary:

..... ..... ..... ..... ..... ..... government funding
..... ..... genetic research :arrow: ..... or
..... ..... ..... ..... ..... ..... corporate funding

Thus:

..... ..... ..... government funding
..... ..... ..... ..... + ..... ..... ..... :arrow: ..... genetic research
..... ..... ..... corporate funding

Again, this is a fact set, so we might expect the Must Be True question which follows. Only one
answer choice will pass the Fact Test, and will be confirmed by the statements above.

Answer choice (A): The stimulus provides that the government is the source of funding for most
genetic research, but the author says nothing about what source of funding more often leads to
resulting advances. Since this choice cannot be confirmed by the facts in the stimulus, we can
confidently rule it out.

Answer choice (B): Like incorrect answer choice (A) above, this choice cannot be confirmed with
the facts provided in the stimulus. There is no information regarding what portion of government
funded genetic advances lead to ethical dilemmas, there is no way to assess the accuracy of this
statement, so it cannot be the correct answer choice.

Answer choice (C): The author tells us that genetic research which is not government-funded is
funded by corporations, but provides no information regarding the success of corporate-funded
research. Because the stimulus provides no confirmation that any such research (funded by
corporations) leads, or has led, to any advances, this choice does not pass the Fact Test.

Answer choice (D): This is the correct answer choice; it can be confirmed with the second
conditional contrapositive diagrammed above, replicated below:

..... ..... ..... government funding
..... ..... ..... ..... + ..... ..... ..... :arrow: no genetic research
..... ..... ..... corporate funding

Common sense dictates that if there is no genetic research there can be no ethical dilemmas which
arise from advances in genetic research:

..... ..... ..... genetic research :arrow: resulting ethical dilemmas

Linking these two statements, we can confirm choice (D) to be the correct answer:

government funding
..... + ..... ..... :arrow: genetic research :arrow: resulting ethical dilemmas
corporate funding

With neither government nor corporate funding, there is no genetic research, and if there is no
genetic research there can surely be no ethical dilemmas arising from advances in genetic research.

Answer choice (E): The author opens the stimulus with the fact that “Almost all advances in genetic
research give rise to ethical dilemmas.” This statement provides no basis for the claim that this trend
will continue indefinitely into the future.
 eober
  • Posts: 107
  • Joined: Jul 24, 2014
|
#16595
Hi,

So the way I have this conditional reasoning is something like this:

Advance in Genetic Research :most: Ethical Dilemma

GR :arrow: Government or Corporation funding

And answer choice D is saying: ED :arrow: G or C

Is this correct way to diagram?

Thanks!
 Nicholas Bruno
PowerScore Staff
  • PowerScore Staff
  • Posts: 62
  • Joined: Sep 27, 2011
|
#16604
I do not think that the first statement is a conditional relationship. In fact the right answer to the question does not require you to diagram that first sentence as a conditional relationship.

Just take the last conditional relationship: GR :arrow: Government or Corporation funding

Answer Choice D is correct because it says that "No ethical dilemmas *resulting from advances in GR* arise without G or C."

This is only saying that *if an ethical dilemma occurs from advances in GR* then *G or C*. Which matches exactly with your conditional relationship GR :arrow: Government or Corporation funding. The first sentence is just extra background information.

Does that make sense?
 eober
  • Posts: 107
  • Joined: Jul 24, 2014
|
#16638
Thank you so much for the explanation again! :)

Is there a strategy that I can follow with MBT questions? I know it is common to see contrapositive as the correct answer choice but before going to the answer choices what do you usually do? Focus on the relationship between the elements in premises and conclusion or something else? (since it is easy to get confused as you are going through the answer choices) Thank you :)
 lsatstudent
  • Posts: 7
  • Joined: May 29, 2016
|
#27717
Hi,

Why isn't C correct? If all genetic research projects are funded by the government or by corporations, and almost all genetic research give rise to ethical dilemmas, then it is true that at least some projects funded by corporations give rise ethical dilemma. Or is this a could be true answer choice?

Wouldn't C be easy to prove from the information provided in the stimulus? :-?

Thanks!
 David Boyle
PowerScore Staff
  • PowerScore Staff
  • Posts: 836
  • Joined: Jun 07, 2013
|
#27737
lsatstudent wrote:Hi,

Why isn't C correct? If all genetic research projects are funded by the government or by corporations, and almost all genetic research give rise to ethical dilemmas, then it is true that at least some projects funded by corporations give rise ethical dilemma. Or is this a could be true answer choice?

Wouldn't C be easy to prove from the information provided in the stimulus? :-?

Thanks!

Hello lsatstudent,

First, a quick note to eober: prephrasing can be valuable, and the "Fact Test" making sure that the answer choice for an MBT question is provable by the facts in the stimulus.
As for Question 15: thinking about the numbers and percentages, we have no way to assume that corporate projects cause advances. --For example, let's say that the government funds 90 percent of genetic research, and corporations fund only 10%. For all we know, the advances occur only in the 90 percent of research funded by the government, not the tiny portion funded by corporations.

Hope this helps,
David
 akanshalsat
  • Posts: 104
  • Joined: Dec 20, 2017
|
#47578
Hello :-D I got this answer correct, and chose D, b/c it made sense to me that if you need gov or corps to fund it, then any advances in general cannot happen without either gov or corps, but I dont truly understand why B is wrong... Thanks!
 Adam Tyson
PowerScore Staff
  • PowerScore Staff
  • Posts: 5153
  • Joined: Apr 14, 2011
|
#47627
The problem with (very attractive) answer B, akanshalsat, is the difference between "most genetic research funded by government" and "almost all advances in genetic research." While we know from the stimulus that most genetic research is funded by government, we don't know the success rate (the percentage of those projects that lead to advances) of those government funded projects. It's only the ones that result in advances that (almost all) give rise to ethical dilemmas.

Imagine, for a moment, some simple numbers (and this is my favorite way to sort through confusing numbers and percentages problems, although it may not be the most efficient way to do it and should be reserved for when you are stuck between contenders or are completely lost in the numerical relationships in the argument). For my analysis, there are 100 total genetic research projects.

Government has to fund most of those - that means at least 51 of those projects get their funds from the government.

Now, how many of those 51 projects lead to advances? Using the information we got from the stimulus... I have absolutely no idea! Maybe none of them? They are all failures? Nowhere did they tell us what percentage of projects result in advances, only that almost all of the ones that DO lead to advances will give rise to ethical dilemmas.

Stop right there, because that's all we need to show that answer B does not have to be true! While most of the projects were funded by government, none of them had to lead to advances, let alone to ethical dilemmas.

Step by step, methodically going through the premises to see what we can prove based upon them, you will find yourself falling short with answer B. The only way to get there is to assume that most of the government funded projects lead to advances.

Tricky stuff, but fun with numbers! That's the stuff I enjoy on this test the most. I hope you have fun with it, too!
 Iam181
  • Posts: 10
  • Joined: Sep 02, 2019
|
#67883
HI!

Is it fair to distinguish genetic research with advance genetic research?

My conditional graph looks like this:

G or C :larrow: GR -?-> AGR :most: ED

GR= Genetic Research

AGR= Advance Genetic Research

-----------------------------------------

Also does "almost all" mean "most" or "95%+" ?

Thank you!
 James Finch
PowerScore Staff
  • PowerScore Staff
  • Posts: 943
  • Joined: Sep 06, 2017
|
#67902
Hi I am 181,

The stimulus is actually talking about "advances in genetic research," which refers to successful genetic research, so it is effectively a subgroup of the larger group of genetic research. And yes, "almost all" is effectively synonymous with all for conditional stimuli, but the because of the group/subgroup issue, the conditional statement given in the first sentence isn't applicable to genetic research as a whole. Instead, we can apply what is true of all genetic research to what would be true of advances in genetic research, although we can't apply what is true to most genetic research to it. The stimulus diagrams out to:

P1: Genetic ResearchAdvances :arrow: Ethical Dilemmas

P2: Genetic Research :most: FundingGovt

P3: Genetic Research :arrow: FundingGovt or FundingCorp

So the only clear inference to make is to combine the first and third premises:

Genetic ResearchAdvances :arrow: Ethical Dilemmas + (FundingGovt or FundingCorp)

In a somewhat convoluted way, (D) is saying exactly this, that all advances in genetic research result in both ethical dilemmas and require funding from either government or corporations.

Hope this clears things up!

Get the most out of your LSAT Prep Plus subscription.

Analyze and track your performance with our Testing and Analytics Package.